User Avatar
megsvyas473
Joined
Apr 2025
Subscription
Free
PrepTests ·
PT114.S4.Q25
User Avatar
megsvyas473
Wednesday, Mar 30 2016

In a could be true EXCEPT (essentially MBF) question, we're looking for something that directly contradicts something that's said in the stimulus. So, for elements that are mentioned in answer choices but NOT mentioned in the stimulus, sure, we could concede these things can be true. a mentions african music, which we know nothing about so sure, this could be true. Similarly, b mentions the military, c mentions chinese music, e mentions something about losing appeal -- all things which aren't in the stimulus which we know nothing about. Only d mentions something mentioned in the stimulus (unintelligible and coherence).

User Avatar

Tuesday, Jul 28 2015

megsvyas473

PT21.S2.Q07 - brownlea's post office

http://classic.7sage.com/lsat_explanations/lsat-21-section-2-question-07/

I initially picked C, probably making some bogus quick logic about how the new bus routes would actually cost the city more and therefore it would not be cheaper to build on the outskirts of town. However, I realize that the answer is B, but still need some more clarification/confirmation in my head to see how that'd be it for sure.

The conclusion is that the Brownlea's post office must be replaced with a larger one. The premises are - the present one cannot be expanded, land near the present one in the city-center is more expensive, and since the acquiring of LAND is where the major cost of a new post office would come from, the outskirts of town is the best place to build it -- because land there must be way cheaper.

B would be the answer because it says that a parking lot would be required if it is built on the outskirts of town and a parking lot would not be required if it is built in the city-center. So, ONE WOULD NEED TO BUY/ACQUIRE MORE LAND (aka more costs) if one builds the post office in the outskirts vs. the city-center. So, building on the outskirts is not necessarily cheaper -- because of the "more" land argument/parking lot which wouldn't be needed if it was built in the city-center.

PrepTests ·
PT120.S4.Q14
User Avatar
megsvyas473
Thursday, Jan 28 2016

Premise: the handwriting is not Miller’s

Conclusion: so, the joker is apparently someone else

Really? Just because the handwriting on the note isn’t Miller’s, can we really conclude that the joker was apparently someone else? What if Miller just got someone else to write the note? Then the joker would be Miller, while the handwriting is still not hers. Or, what if she had an accomplice? Then the joker could still be Miller, but her accomplice (also a joker) may have written the note instead

a- it fails to consider the possibility that there was more than one joker. If there WAS more than one joker, then the conclusion that the the joker was someone else (besides Miller) doesn’t necessarily follow – the joker could have still been Miller (and she could have other accomplices who wrote the note)

b- it’s not logically required to indicate the degree to which handwriting samples have to match

c- argument doesn’t need to provide an explanation why Miller would be the prime suspect. That is not the flaw in how the premises (handwriting not hers) and the conclusion (joker is not Miller, it’s someone else) connect

d- not needed to provide an explanation about this

e- it does take this for granted, it’s likely that if the handwriting matched, Franklin would have ascertained identity to Miller, but this is not the flaw in how the premises support the conclusion – aka, just because the handwriting on note doesn’t match Miller does NOT mean that Miller is not the joker

PrepTests ·
PT120.S4.Q1
User Avatar
megsvyas473
Thursday, Jan 28 2016

The paradox is: big budget movies make A LOT of money, but the movie industry gets MOST of its revenue from low-budget movies.

We need something that explains this -- how come we get most of our money from low-budget movies? We are ideally looking for something that low-budget movies have (like quantity) that big-budget movies don't have. This could help EXPLAIN the discrepancy.

a- okay, big budget need to sell more tickets. Cool. I still don't know why we get most of our money from low-budget movies

b- there are many more low-budget movies overall than any other type of movie. This -- far greater quantity -- could help explain WHY we get most of our revenue from low-budget movies. If we make just one big budget movie per year, sure it makes A LOT, but if we make a hundred thousand more low-budget movies then that might be why we get MOST of our revenue from low-budget movies

c- okay, this just tells me that overall the movie industry's revenues have declined. It doesn't give me a reason why we get most of our revenue from low budget movies

d- this just tells me that big-budget movies cost more to make. SO WHAT? The fact still remains -- they make A LOT. It wouldn't give me a reason why low-budget movies give us the most revenue.

e- this just gives me a random fact that time is tied to cost. I cannot and should not make any other assumptions in regard to this fact.

PrepTests ·
PT120.S4.Q11
User Avatar
megsvyas473
Wednesday, Jan 27 2016

Premise: many psycholgists would disapprove of this traditional childrearing practice because they believe it damages a child’s self-esteem.

Conclusion: however, it is fact that adults raised under this traditional child rearing practice are, on average, AS CONFIDENT AS (equally confident) as adults that weren’t raised under this practice.

e- if children’s loss of self-esteem makes them less confident as adults (as the psychologists say), then the traditional child-rearing practice did not tend to cause SIGNIFICANT loss of self-esteem. Yes! Because how else could we conclude that those raised under this traditional practice AND those not raised under it have equal confidence? Notice we're talking about averages. It would have to be the case that those raised under this practice did not have a significant loss of self esteem (and this did not have a significant loss of confidence), otherwise, there’s no way we can conclude/agree it’s a fact that adults under this practice and not under this practice have equal confidence.

a- we can’t outright say that their beliefs are incorrect, their beliefs may very well be correct, it’s just that some of the “average” data just tells us that both under trad. practice and outside of it are of equal confidence – that doesn’t mean these psycholgists are incorrect, maybe there is some other element at play, which makes it so that although the psychologists ARE correct, it doesn’t apply in this particular case

b- we don’t know if some of the “most” confident adults and the “least” confident adults were raised under the trad. practice in question. We just know that these adults are AS confident as those not raised. That’s all – their relative, “very” or “not very” confidence level is not something we know of because it's an average.

c- no, irrelevant

d- we have no indication of this, there’s nothing that allows us to conclude this

PrepTests ·
PT119.S4.Q23
User Avatar
megsvyas473
Wednesday, Jan 27 2016

Argument says:

exercise vigorously is correlated w/ more healthy, but this does not mean that exercise vigorously CAUSES more healthy. In fact, it’s the reverse; more healthy (pre-existing state of health) CAUSES exercise vigorously.

Argument presents us with a correlation, and alerts us that this does not mean causation, but then says it means another causation.

a- habitual readers are correlated w/verbally skilled, but this does not mean that reading CAUSES verbal skills, in fact it’s the reverse; verbal skill CAUSES reading.

b- Some mathematicians and equating that with skilled musicians throws this one out – we’re looking for correlation and causation, this mixes musicians and mathematicians

c- There is no reverse causation in this. In order for this argument to work, it would have had to say something like: people’s dressing habits are correlated with their friends, this does not mean that dressing habits cause friends (dressing habits cause you to pick friends), in fact, it’s the reverse, friends cause your dressing habits. As this answer choice stands, it just gives us a correlation, and then tells us “this does not mean causation” and then stops short.

d- This is wrong because it introduces correlation, says it doesn’t mean causation and then introduces a “frequency” element. As it stands, it says: taller children are correlated with outperformance, this doesn’t mean that “tallness” causes “outperformance” – it should then say, in fact, outperformance causes tallness – but instead, it says because taller children tend to play MORE FREQUENTLY – which also crushes the correlation and causation, but doesn’t introduce a reverse causation like the stimulus.

e- No.

PrepTests ·
PT119.S4.Q17
User Avatar
megsvyas473
Wednesday, Jan 27 2016

#17, weaken

those who drink soft water have an increased risk of disease. Because those who have these diseases have lower levels of magnesium in their blood, and hard water contains more calcium and magnesium.

Basically argument is that soft water (less magnesium) causes these diseases.

e- gives us a weakener to this causal relationship, it tells us that actually the meds that people with these diseases take diminish the body’s ability to absorb and retain magnesium. So, it’s NOT that less magnesium causes these diseases but that the medicines that treat these diseases CAUSE less magnesium. So, we can’t make the link that soft water (less magnesium) causes these diseases – since we now know that soft water/less magnesium doesn’t cause these diseases, but rather the meds from these diseases cause less magnesium.

a- this is just a random fact about magnesium deficiency, it doesn’t touch our argument of SOFT WATER/LESS MAGNESIUM CAUSES THESE DISEASES at all.

b- Random fact

c- Random fact, irrelevant

d- This simply tells us that magnesium vitamins (more magnesium) inhibits the effectiveness of SOME medicines that treat these diseases. So what?? This is a random fact that doesn’t touch our argument that “less magnesium causes diseases”—instead this answer choice basically says “more magnesium hurts some medicines that treat these diseases” okay, medicines are irrelevant to the argument. With this answer choice, less magnesium/soft water can still CAUSE these diseases.

PrepTests ·
PT119.S4.Q15
User Avatar
megsvyas473
Wednesday, Jan 27 2016

Marta: There have been some complaints about the lack of recreational areas in our city. Some people favor turning the old railway land into walking trails, but there may be more productive ways of using that land (Marta’s conclusion).

Arthur: But the old railway land is ideal for walking trails. We should not dismiss this proposal for walking trails without further consideration.

Two things: just because Marta thinks may be more productive uses of the land doesn’t mean she doesn’t think that land is ideal for that purpose. Also, Marta is NOT dismissing the proposal for walking trails, she’s simply introducing a reason why perhaps we shouldn’t use the old railway land to do this.

Notice the dialogue exchange, Marta says “there MAY BE (not certain but MAY BE) more productive ways of using that land, and then Arthur comes in and says “we should not dismiss this proposal for walking trails without further consideration” – basically, he assumes that she is completely dismissing this proposal without further consideration, which she is not doing. She’s simply introducing a reason why perhaps the old railway trail MIGHT NOT BE a good place to have a walking trail, she’s simply giving a reason why we should reconsider using that particular piece of land for that purpose, it’s not an outright dismissal, as Arthur takes it to be.

a- Marta does not say that this would be an ENTIRELY unproductive use of the land (notice her wording “may be more productive uses), and moreover, Arthur also doesn’t take her to be saying this – when she says there may be more productive uses of that land, he immediately responds “we should not dismiss it without further consideration” – so he takes her thought of “there may be more productive uses” as a “dismissal” of the proposal, otherwise, there is no reason he would say “we should not dismiss this proposal without further consideration”

b- No

c- No

d- Yes, he responds “we should not dismiss it without further consideration” – he believes her suggestion that there may be more productive uses of the land to be dismissing the proposal promptly

e- No indication that Arthur thinks Marta thinks it’s not possible

User Avatar
megsvyas473
Wednesday, Jan 27 2016

Okay, cool!

PrepTests ·
PT119.S4.Q14
User Avatar
megsvyas473
Wednesday, Jan 27 2016

Premise: if on major issues media was narrower --> critics claim true, but media is not narrower on major issues

Conclusion: so, critics claim is false

This is a classic flaw, where they fail the sufficient condition and declare the failure of the necessary condition, which we know is something you cannot do!

c - argument takes for granted that if the truth of one claim (media narrower) implies the truth of another claim (critics claim is true), then the falsity of the first claim (media not narrower) implies the falsity of the second claim (critics claim not true)

a- there's no personal attack

b- argument doesn't take for granted that the media should give any set amount of exposure to anything

d- no appeal to popular opinion is made

e- doesn't take for granted that any certain range or type of opinion is desirable

User Avatar
megsvyas473
Wednesday, Jan 27 2016

@ I thought PT 42 was on Saturday? Ahh, trying to keep up lol

User Avatar
megsvyas473
Wednesday, Jan 27 2016

Hahahah nice!!

PrepTests ·
PT119.S4.Q10
User Avatar
megsvyas473
Wednesday, Jan 27 2016

It's very important to notice the subtlety in Olaf's answer in order to notice exactly which part of Charlene's argument he THINKS he's attacking. Olaf misinterprets Charlene's use of the term "normal" -- Olaf understands it to be overall relative normal, while Charlene is using it to refer to what is "normal" for the region.

Charlene says -- environmental cleanup's reliance on microbes has its limitations because microbes become less active when a region's temperature drops below normal.

Olaf disagrees and gives examples of two regions -- Arctic and subtropical, where although the temperature is known to be VERY ABNORMAL COMPARATIVELY to the rest of the regions of the world, the microbes still eat oil/cleanup at nearly the same rate. So basically Olaf says -- hey, no, here's two regions where temps are not normal and microbes are not any less active. The issue in Olaf's reasoning is that Charlene is talking about when a regions's temps drop below normal (that's when microbes become less active), whereas Olaf gives examples of regions where temps are technically comparatively abnormal, but they ARE NOT "dropped below normal" for the region. We need to see the distinction between dropping below normal for the region (and that resulting in less active microbes) versus abnormal temps in general. They are not the same thing, and we shouldn't expect places that have abnormal temps in general to necessarily have less active microbes (because they don't have "temps dropping below normal for the region).

PrepTests ·
PT131.S3.Q13
User Avatar
megsvyas473
Tuesday, Jul 26 2016

This question is about frequency. If a large car is going to get into a lot more accidents, then a low percentage of a lot more accidents might actually mean that one maybe isn't less likely to be injured in an automobile accident if one drives a large car.

Say, for example, that 9,000 large cars got into accidents, and 10% of those (what is considered a "low percentage") got injured -- still that's like 900 injuries.

And at the same time, only 1,000 small cars got into accidents and 50% (what's considered a "high percentage") of those got injured -- so that's like 500 injuries.

Still, this wouldn't "prove" that you're less likely to be injured in a large car, because, HELLO, there were 900 injuries in those large cars!! The issue is that of these 10,000 people involved in accidents, we just don't know how many (aka the frequency) of them in their respective, small or large cars were involved in the accidents in the first place, so it's hard to make any conclusions about likelihoods of injuries without that information of HOW MANY were involved/frequency of the accidents in the first place.

PrepTests ·
PT102.S4.Q16
User Avatar
megsvyas473
Saturday, Mar 26 2016

logically, "one but not both" means (/A or /B) and (A or B)

So let's say, to get to class on time, I need an umbrella or a poncho but not both. Logically that means: 1) umbrella and not poncho or poncho and not umbrella.

The negation of "one but not both" is" A and B & /A and /B. So, if I had both poncho and umbrella, that violates the condition, and if I had neither poncho nor umbrella then that also violates it.

In #25 -- the stimulus says "the people I meet have one or the other of these qualities but not both." Whereas, (D) says "no one is either wise or intelligent" (no one is either of those qualities) which logically means: everyone is /A and /B, which is a direct negation of "one but not both."

PrepTests ·
PT115.S4.Q23
User Avatar
megsvyas473
Monday, Jan 25 2016

#23, flaw

Premise: if maximum total utility is ASSURED then it’s a pure free market economy (MTUA → PFME), aka pure free market economy is a necessary condition to the ASSUREMENT of maximum total utility. There are other economies that might be able to achieve maximum total utility, though.

Conclusion: if you aren’t trying to achieve about pure free market economy then you are not acting in a way that is MOST LIKELY to bring about maximum total utility.

What? The argument simply says that when maximum total utility is ASSURED, then a requirement to that is pure free market economy. There are other ways/economies that bring about maximum total utility, and some of them actually might be MORE LIKELY to bring about maximum total utility.

Just because pure market economy is a necessary condition to when maximum total utility is assured, does not mean that it is the most likely way to bring it about this is what (d) is getting at

a-does not presume that if you don’t have this one economy, then you have this other economy, no such indication that it’s just one or the other

b-doesn’t presume any such thing

c-argument doesn’t say that “this is the most likely way to achieve this end” and it’s the “only way” to achieve this end – it does not make this sort of jump – it never says pure free market is the most likely way in the first place, and thus cannot say that it’s the “only way”

d-yes – presumes without justification that trying to bring about a condition that will ensure (pure free market) that will ensure the achievement of an end (maximum total utility) must be the way most likely (thus pure free market is the most likely way) to achieve that end

aka just because pure free market is a necessary condition to the ensure of an end, it does not mean it’s the most likely way to bring about that end

e-doesn’t need to consider this possibilty

PrepTests ·
PT115.S4.Q19
User Avatar
megsvyas473
Monday, Jan 25 2016

#19, necessary assumption

something that absolutely has to be true for the argument to be valid

Conclusion: it’s obvious one ought to have a will stating how one wishes one’s estate to be distributed

Premise: without a will, distant relatives you’ve never met have a greater legal right to one’s estate than beloved friends do

a- no one? It’s not necessary to the argument that NO ONE at all, ever, would want this

b- only? Does it have to go ONLY to a person who is deserving? This wouldn’t be necessary to the argument, also we don’t know who is more deserving in the scenario (maybe the relative is because they are “blood”)

c- are unjust? It is not necessary to the argument that they have to be unjust. It’s just that they aren’t the way some people would “prefer” them to be, but justice (or not) is not part of the argument

d- if we negated this, aka if people WERE generally (mostly) indifferent about how their estates were distributed, then it would be very hard to support an argument that says “you OUGHT to have a will stating how your estates should be distributed” – why would I ought to have this if I’m indifferent to what happens to my estate?

e- Not necessary for one’s beloved friends to have a GREATER LEGAL RIGHT. It could just be some other right, or something else, anything else, something that maybe makes them more entitled. Also, this answer choice makes it seems like they have a greater legal right in every instance, which is just not necessary to the argument. Some people might be okay with distant relatives having a greater legal right to their estate.

PrepTests ·
PT115.S4.Q16
User Avatar
megsvyas473
Sunday, Jan 24 2016

a-can’t say it’s “highly” controversial based on “ a small few reject this idea” notice the definitiveness of the word “IS” highly controversial

b- we don’t know what type of artist Cezanne WAS, there’s clearly a little debate around it; notice the definitiveness of the word “was” – we just can’t say for sure

c-Cezanne “helped to develop modernism” – this is nice and meek, sure, we can say he “helped” to develop “modernism” – we know, for a fact, that he “inspired” the next generation of modernist creators of abstract art, so in some way his “inspiration” helped to develop the art of the artists he inspired

d- we don’t know what many experts think about what modern art “owes” to Cezanne, to think we know this would be over-reading. Is there any indication in the stimulus that’s like: “oh modern art owes a lot more to Cezanne than he is getting” to which we can then say that modern art owes less to him than these experts believe? No.

e- misinterpreted as modernist? We don’t know which interpretation is “correct.” We just know that there are at least two interpretations of his work, we don’t know if either of them is a misinterpretation.

User Avatar

Wednesday, Jul 22 2015

megsvyas473

P22.S4.Q12 - most adults in country x

http://classic.7sage.com/lsat_explanations/lsat-22-section-4-question-12/

This was confusing! Adults in Country X consume an increasing amount of fat as they grow older, but the percentage of fat in their diet stays the same throughout adult life. Initially, I approached the answer choices thinking: "okay, this is answer going to disqualify a misconception on numbers and percentages" or something along those lines. However, I didn't find this to be the case. The answer is (B), and it says that they generally EAT MORE when they are older than they did in their earlier adult life. I can't help but think, "uh, we don't really know this, do we?" Yes, they might EAT MORE (meaning the quantity might be more), but what does that have to do with them maintaining the percentage of fat in their body? I mean it could technically have no effect, right? Help, please! :)

http://classic.7sage.com/lsat_explanations/lsat-19-section-4-question-24/

Hi friends! I want to make sure that my reasoning is on the ball for this question, since I got it wrong the first time around.

Clarification before you read on: everything in parenthesis is taken from the stimulus, and anything not in parentheses is from the respective answer choice. I mostly need help distinguishing more clearly, A, D, and E from each other.

(A): At least one of the players in the orchestra (at least one of the joists in the wall) must have made a mistake (must have broken), since nothing else would have made the conductor grimace in the way she just did (since nothing else could have caused a bulge).

(B) doesn't match up a "fault" or a "cause" with why something must have occurred, and there is no "at least" element, either; (C) this seems pretty out of scope, especially with the X do Y only when Z

(D): This is the answer I incorrectly picked at first, with some hasty logic like: "Oh, if they are playing a piece of music that has a harp (if there is a bulge), then at least one of the players must play the harp (just like at least one of the joists must be broken). VERY BAD, I know, and answer choice (A) matches up much better. Not to mention, that (A) correctly matches the stimulus in its noting of "at least" and (D) notes "one" -- which is not a correct match.

(E) this also seems out of scope, but has those tricky, creepy, psych elements that JY talks about. "The emotion of the music is the only thing" (could be equated with the joist being the only thing), that could have caused the conductor to look so angry (that could have caused the bulge) just then, since the orchestra was playing perfectly. SEE, here there is no parallel available for the "just then" and the "since the orchestra was playing perfectly" - there is no "the wall was fine and then suddenly there was a snap noise" and so "just then, the joist could have been the only thing for the bulge," etc.

Am I making any sense?

I'm usually totally fine with these types of questions. Any thoughts?

User Avatar

Friday, Dec 18 2015

megsvyas473

Really need study buddy

Hello my fellow 7Sagers, I'm studying for the June LSAT, and am in desperate need of a study buddy. I'd love to keep each other accountable (more or less according to the June BR group schedule graciously made by @DumbHollywoodActor) and work through concepts/teach other stuff/BR together. I think Skype would be a great forum for us to do this (unless we somehow freakishly live near each other). Please, please reach out to me if you think this is something you'd like to do! Thanks!

User Avatar
megsvyas473
Wednesday, Feb 17 2016

Same here!

User Avatar

Friday, Oct 16 2015

megsvyas473

3 month PT schedule, any advice?

Hey all --

I'm planning on taking the Feb test (though I'm not against pushing it to June). I've finished the Ultimate curriculum thoroughly, gone through the Trainer and am currently working on fool proofing games 1-35 using Pacifico's amazing LG games "4 times" strategy. I'm also drilling LR and RC sections here and there to keep those skills fresh as I work on perfecting my LG. I plan to finish the 1-35 games by the end of October and then start a PT schedule.

Is there an ideal PT schedule any of you would be willing to share with me, particularly one that includes 36-75 and mixes them up in a nice order; if you guys recommend progression from 36 to 75, I'm fine with that as well - I just need some wonderful mentor advice here. Also, I'd like to throw in that I'm a little nervous whether or not 12 weeks is enough to PT and BR all those tests -- please feel free to share experiences on this as well. I'm really okay with pushing to June, if it's recommended. Thanks in advance!!

PrepTests ·
PT127.S3.Q10
User Avatar
megsvyas473
Friday, Jul 15 2016

Analogous argument: I did just as well as Stacey in my marathon split time, so I did above average.

Um, wait? Did Stacey do above average? The argument is leaving out a premise/making an assumption that Stacey did in fact do above average (aka, it takes this for granted).

User Avatar

Monday, Oct 05 2015

megsvyas473

Study strategy advice moving forward

Fellow 7Sagers, I need some help on proceeding how to move forward with my prep. I just recently finished the curriculum and took June '07 and PT20 as timed PTs. I did thorough BRs on both of them - first June '07 and then PT20. My main concern/issue was how much I actually remembered of the PT20 LR from the curriculum, and honestly felt that this could not possibly be representative of an actual PT. Before any of this, I initially thought to myself "I'm going to PT every single test 1-35 and BR thoroughly and then move forward to 35+," but I realize this would not be conducive, and is probably too rigid of a mindset. I can't possibly take 1-35 as timed PT takes, I need to build stamina AND have material too drill, rather than just doing PT takes.

My strategy moving forward is below, but please feel free to help me alter it. I don't want to touch PT35+ until I have touched and done BR on the PTs below 35. Is this a fair way to approach it?

-Use PT 15-30 to drill timed sections, slowly building up stamina (by increasing section takes) and do a thorough BR on all sections (and obv fool proof LG) -- the reason I thought these would be best for timed drilling is because a lot of this material appears in the curriculum.

-Take 1-14 and 31-35 as timed PTs, and do thorough BR on them (fool proofing LG). This will give me practice on timed tests, after I've built up stamina and done BR on other material.

My issue is: am I trying to be too methodological about this? Should I just drill timed sections, and then move forward with PTing 35+? This is especially relevant to LG -- I want to do a "finish the bundle"-like approach -- maybe I really should just work on fool proofing all games from 1-35 and drilling the other remaining sections on there and then move forward?

I'd love any help/advice!!!

PrepTests ·
PT116.S2.Q16
User Avatar
megsvyas473
Thursday, Feb 04 2016

BECAUSE people like to live in regions of natural beauty, businesses are attracted to relocate to these regions (of natural beauty).

THUS, gov. mandated env. protection in regions of natural beauty can help those economies OVERALL, even if such protection harms some older local industries.

Some assumptions: 1) government mandated protection WILL actually encourage business to relocate those these regions (and not scare them away), 2) the harm caused to local industries won't be to the extent that NEW BUSINESSES will not want to relocate there

b - if the economies of natural beauty regions WERE based primarily on local industries, and the env. reg. would harm them -- it wouldn't really touch our argument -- which revolves around the question whether NEW BUSINESSES would still be encouraged to come there. Because even if local industries were the primary center of these regions and were destroyed because of the mandate, but the mandate still brought in new businesses, we can STILL CONCLUDE that these new businesses "will help the economy OVERALL"

e - this is an assumption that is necessary, if the mandate discourages businesses from relocating to that region, then we cannot say that the mandate "will help economies overall" -- because then clearly, what is supposed to help the economy overall, is actually harming it.

User Avatar
megsvyas473
Wednesday, Feb 03 2016

@ the crew missed you today!!

PrepTests ·
PT124.S2.Q19
User Avatar
megsvyas473
Tuesday, Feb 02 2016

a - locals wouldn't have to spend anything to TRAVEL to and from local dealerships (whereas tourists would), so yes, it's worth it for locals

b - it's more worth it for tourists to go with national firms because they DO NOT need to spend money to TRAVEL there (as they would for local dealerships)

c - tourists don't know which local dealerships offer the discounts, and they can't find out easily from another way either (like say a tour guide)

d - tourists don't know of local dealerships that offer rentals, but locals DO know of these (aka, locals wouldn't have to spend to get this information, they already know, so local dealerships is worth it for them)

e- "no less expensive" means that it actually could cost the locals MORE MONEY to travel to local dealerships, which is bad, this would actually weaken the argument. If it had said it would be "no more expensive" for locals to get to local dealerships than national rental firms, then that would strengthen our argument. That would guarantee that it would cost locals the same to get to both, and so local dealerships are more worth it.

EDIT: actually (e) just tells us that it costs the same for locals to get to both locations/dealerships, so it would do nothing for our argument.

PrepTests ·
PT122.S1.Q16
User Avatar
megsvyas473
Monday, Feb 01 2016

#16, weaken

the analysts claim that prices on rice increased because government growers control most of the rice and distribute it for local consumption. Because so little rice is traded freely, and only a small percentage is sold commercially, these government growers are influencing/making price increases on rice worse during these times of decreased production.

A- yes, that makes sense. They would likely do that, but this doesn’t weaken the argument of the analysts that: because government growers control most of rice by distributing for local consumption, price of rice remains high.

B- this somewhat support the analysts’ claim – government growers are now perhaps controlling rice even more “store more of the rice they control”

C- so the analysts are saying – BECAUSE government growers control most rice and distribute for local consumption, the price of rice in world markets is high. This answer choice says that actually, in these times (of decreased rice production), government growers don’t do this so strongly – some of the rice that they intended to distribute locally (one way of control), they export that rice to countries with free grain markets, so they’re actually sharing their “share” of the rice with others, so they aren’t necessarily ENTIRELY controlling and influencing rice price increase

D- this might strengthen analysts’ claim because it tells us that the governments who control (by distributing for local consumption) actually also come and PURCHASE the rice that is sold commercially, which is already available at such a limited percentage – so actually they REALLY control now

E- irrelevant

PrepTests ·
PT122.S1.Q8
User Avatar
megsvyas473
Monday, Feb 01 2016

Let's find a REASON WHY vervet monkeys have different/distinguishing alarm calls based on WHERE the predator is coming from.

WHY do they use different calls based on where the predator comes from? Maybe they're indicating to monkeys which TYPE of animal is approaching, or telling them where they should hide/take cover.

a - number of predators approaching? We're not looking for number. WHY do they distinguish BASED ON TYPE of predator, land or air. Number won't explain why different calls depending on where predator coming from.

b - random.

c - that's great. I'm glad there are no freaky duo-possibility animals out there that attack these monkeys. But this would not give us a reason WHY they use different/distinguishing calls based on WHERE the predator is coming from.

d - explains WHY distinguishing alarm calls are used. Because they monkeys are telling their friends WHERE exactly the danger is coming from, and accordingly, WHERE/HOW they should take cover.

e - cool, wouldn't explain why different/distinguishing calls.

PrepTests ·
PT122.S1.Q7
User Avatar
megsvyas473
Monday, Feb 01 2016

To be an immoral action, it is necessary that you are accompanied by feelings of guilt. IA --> FG. It is a necessary condition to be an immoral action that you are accompanied by feelings of guilt. So, we cannot say (a), that there is an immoral action that is NOT accompanied by feelings of guilt. That would just be a contradiction.

(b), sure could be true

(c), (d), and (e) are all talking about actions in general. The stimulus gives us conditions for an immoral action and we need to find an answer choice that directly clashes/contradicts with the information given to us in the stimulus.

Confirm action

Are you sure?